Sr Examen

Otras calculadoras:

Límite de la función (n^2+n/log(n))/(n^3-(-1+n)^3)

cuando
v

Para puntos concretos:

Gráfico:

interior superior

Definida a trozos:

Solución

Ha introducido [src]
     /  2     n     \
     | n  + ------  |
     |      log(n)  |
 lim |--------------|
n->0+| 3           3|
     \n  - (-1 + n) /
$$\lim_{n \to 0^+}\left(\frac{n^{2} + \frac{n}{\log{\left(n \right)}}}{n^{3} - \left(n - 1\right)^{3}}\right)$$
Limit((n^2 + n/log(n))/(n^3 - (-1 + n)^3), n, 0)
Método de l'Hopital
En el caso de esta función, no tiene sentido aplicar el Método de l'Hopital, ya que no existe la indeterminación tipo 0/0 or oo/oo
Gráfica
Respuesta rápida [src]
0
$$0$$
A la izquierda y a la derecha [src]
     /  2     n     \
     | n  + ------  |
     |      log(n)  |
 lim |--------------|
n->0+| 3           3|
     \n  - (-1 + n) /
$$\lim_{n \to 0^+}\left(\frac{n^{2} + \frac{n}{\log{\left(n \right)}}}{n^{3} - \left(n - 1\right)^{3}}\right)$$
0
$$0$$
= -1.82818376864644e-5
     /  2     n     \
     | n  + ------  |
     |      log(n)  |
 lim |--------------|
n->0-| 3           3|
     \n  - (-1 + n) /
$$\lim_{n \to 0^-}\left(\frac{n^{2} + \frac{n}{\log{\left(n \right)}}}{n^{3} - \left(n - 1\right)^{3}}\right)$$
0
$$0$$
= (2.63129599939591e-5 + 1.08437978758584e-5j)
= (2.63129599939591e-5 + 1.08437978758584e-5j)
Otros límites con n→0, -oo, +oo, 1
$$\lim_{n \to 0^-}\left(\frac{n^{2} + \frac{n}{\log{\left(n \right)}}}{n^{3} - \left(n - 1\right)^{3}}\right) = 0$$
Más detalles con n→0 a la izquierda
$$\lim_{n \to 0^+}\left(\frac{n^{2} + \frac{n}{\log{\left(n \right)}}}{n^{3} - \left(n - 1\right)^{3}}\right) = 0$$
$$\lim_{n \to \infty}\left(\frac{n^{2} + \frac{n}{\log{\left(n \right)}}}{n^{3} - \left(n - 1\right)^{3}}\right) = \frac{1}{3}$$
Más detalles con n→oo
$$\lim_{n \to 1^-}\left(\frac{n^{2} + \frac{n}{\log{\left(n \right)}}}{n^{3} - \left(n - 1\right)^{3}}\right) = -\infty$$
Más detalles con n→1 a la izquierda
$$\lim_{n \to 1^+}\left(\frac{n^{2} + \frac{n}{\log{\left(n \right)}}}{n^{3} - \left(n - 1\right)^{3}}\right) = \infty$$
Más detalles con n→1 a la derecha
$$\lim_{n \to -\infty}\left(\frac{n^{2} + \frac{n}{\log{\left(n \right)}}}{n^{3} - \left(n - 1\right)^{3}}\right) = \frac{1}{3}$$
Más detalles con n→-oo
Respuesta numérica [src]
-1.82818376864644e-5
-1.82818376864644e-5